Answer to Question #103444 in Statistics and Probability for Michael james

Question #103444
Assume that a given population is normally distributed with the population mean meu and variance sigma² and that the sample given is chosen sufficiently large such that (x-bar-meu/sigma/√n)~meu(0,1). Show that the Comfidence interval at alpla=0.01 probability is theta1<=meu<=theta2 where theta1=x-bar-2.58(sigma/√n) is the lower C.I and theta2=x-bar+2.58(sigma/√n) is the upper C.I
1
Expert's answer
2020-02-20T09:57:11-0500

A "100(1-\\alpha)% confidence interval" confidence interval


"P(\\hat{\\theta_L}<\\theta<\\hat{\\theta_R})=1-\\alpha"

Writing "z_{\\alpha\/2}" for the z-value above which we find an area of "\\alpha\/2" under the normal curve, we can see


"P(-z_{\\alpha\/2}<Z<z_{\\alpha\/2})=1-\\alpha"

where


"Z={\\bar{x}-\\mu \\over \\sigma\/\\sqrt{n}} \\sim N(0, 1)"

Hence


"P(-z_{\\alpha\/2}<{\\bar{x}-\\mu \\over \\sigma\/\\sqrt{n}}<z_{\\alpha\/2})=1-\\alpha"

"P(\\bar{x}-z_{\\alpha\/2}{\\sigma \\over \\sqrt{n}}<\\mu<\\bar{x}+z_{\\alpha\/2}{\\sigma \\over \\sqrt{n}})=1-\\alpha"

The z-value leaving an area of 0.005 to the right, and therefore an area of 0.995 to the left, is "z_{0.005}=2.58"

Therefore confidence interval at "\\alpha=0.01"


"CI=(\\bar{x}-2.58{\\sigma \\over \\sqrt{n}}, \\bar{x}+2.58{\\sigma \\over \\sqrt{n}})"




Need a fast expert's response?

Submit order

and get a quick answer at the best price

for any assignment or question with DETAILED EXPLANATIONS!

Comments

Assignment Expert
21.02.20, 15:50

Dear Israel Robert, You are welcome. We are glad to be helpful. If you liked our service, please press a like-button beside the answer field. Thank you!

Israel Robert
20.02.20, 22:34

Thanks alot for the help.

Leave a comment

LATEST TUTORIALS
New on Blog
APPROVED BY CLIENTS